6
$\begingroup$

[MOVED HERE FROM MSE.]

The statement of the Grothendieck-Lefschetz fixed point theorem is well-known. For a proper algebraic variety $X$ over $\mathbb F_q$, $$\#X(\mathbb F_q) =\sum_i (−1)^i Tr(Fr_X, H^i_c(X, \mathbb Q_l)).$$ Also known is the version for general constructible l-adic sheaves $\mathcal F$: $$\sum_{x\in X(\mathbb F_q)} Tr(Fr_x,\mathcal F_x)=\sum_i (−1)^i Tr(Fr_X, H^i_c(X, \mathcal F)).$$ Thirdly, K. Behrend proved an analog for the first formula in the context of algebraic stacks (replacing the scheme $X$ by a Noetherian algebraic stack $\mathcal X$).

Now my question is: is there a version of the second formula for an algebraic stack $\mathcal X$ (with nice hypotheses if necessary)?

It would seem natural, since the second formula is a generalization of the first, and the first is true in the context of algebraic stacks by Behrend's work. However, the second formula does not follow directly from the first in the case of schemes (as far as I know: I would be glad if it were true!), so I am not in the position to easily extend the proof of the second formula in the more general context of stacks.

Thank you in advance.

ADDED QUESTION: Moreover, why is the sum on the left of the second formula finite when the scheme (or stack) is not of finite type? Behrend speaks about this problem, but I do not find where he solves it, if he does.

$\endgroup$
3
  • 1
    $\begingroup$ The case of general constructible coefficients is not in Behrend's original 1993 paper as you say. But it's in his 2003 book "Derived $\ell$-adic categories for algebraic stacks". (Also, you don't need $X$ proper in your formula.) $\endgroup$ Jul 7, 2019 at 20:06
  • 1
    $\begingroup$ For the added question: outside the finite type case there's no reason for anything to converge. Behrend does discuss convergence (in the finite type case) but the problematic part is the right hand side, not the left hand side. Indeed a finite type Artin stack can have nonzero rational cohomology in infinitely many degrees. $\endgroup$ Jul 16, 2019 at 8:04
  • $\begingroup$ Here is link to the question on Mathematics: Generalized Behrend version for Grothendieck-Lefschetz trace formula. (It is recommended to link each copy to the other ones when cross-posting.) $\endgroup$ Jul 16, 2019 at 8:08

1 Answer 1

10
$\begingroup$

This is Theorem 4.2 of Shenghao Sun's paper $L$-Series of Artin stacks over finite fields, Algebra & Number Theory 6 (2012) pp 47–122, doi:10.2140/ant.2012.6.47, arXiv:1008.3689.

Let $f:\mathscr X_0\to\mathscr Y_0$ be a morphism of $\mathbb F_q$-algebraic stacks, and let $K_0\in W^{-,stra}_m(\mathscr X_0,\overline{\mathbb Q}_{\ell})$ be a convergent complex of sheaves. Then

(i) (Finiteness) $f_!K_0$ is a convergent complex of sheaves on $\mathscr Y_0,$ and

(ii) (Trace formula) $c_v(\mathscr X_0,K_0)=c_v(\mathscr Y_0,f_!K_0)$ for every integer $v\ge1.$

Here $c_v$ is the sum of the trace of the $v$th power of Frobenius. Applying this in the case where $\mathscr Y_0$ is a point gets you what you want.

$\endgroup$
7
  • $\begingroup$ To amplify on Will's answer, the published version of the paper he mentions is projecteuclid.org/euclid.ant/1513729758 $\endgroup$
    – user141204
    Jul 7, 2019 at 14:24
  • $\begingroup$ @Alex There are comments on now deleted questions (that 10k+ rep users can still see) pointing to various patterns of behaviour involving this user. There is discussion here: meta.mathoverflow.net/questions/4200/flood-of-similar-new-users $\endgroup$
    – David Roberts
    Jul 12, 2019 at 9:35
  • 1
    $\begingroup$ Also, the old Lauziéres account(s) had a history of good interaction, the new account suddenly started asking poorly-motivated questions along the line of the sorts of questions the known sockpuppet accounts used. $\endgroup$
    – David Roberts
    Jul 12, 2019 at 9:40
  • $\begingroup$ I cannot find in Behrend's paper the fact that the two sides of the formula converge. The sum is an infinite sum (varying over every rational point of the stack), so why does it converge? In the 1993 paper Behrend comments this fact, but in the later paper I cannot find anything of the kind... $\endgroup$
    – W. Rether
    Jul 15, 2019 at 16:18
  • 1
    $\begingroup$ @W.Rether The stacks in question have finite type, I believe, making the sum finite. $\endgroup$
    – Will Sawin
    Jul 15, 2019 at 18:46

Your Answer

By clicking “Post Your Answer”, you agree to our terms of service and acknowledge you have read our privacy policy.